Last visit was: 25 Apr 2024, 05:07 It is currently 25 Apr 2024, 05:07

Close
GMAT Club Daily Prep
Thank you for using the timer - this advanced tool can estimate your performance and suggest more practice questions. We have subscribed you to Daily Prep Questions via email.

Customized
for You

we will pick new questions that match your level based on your Timer History

Track
Your Progress

every week, we’ll send you an estimated GMAT score based on your performance

Practice
Pays

we will pick new questions that match your level based on your Timer History
Not interested in getting valuable practice questions and articles delivered to your email? No problem, unsubscribe here.
Close
Request Expert Reply
Confirm Cancel
SORT BY:
Date
Tags:
Difficulty: Sub 505 Levelx   Weakenx                           
Show Tags
Hide Tags
Intern
Intern
Joined: 04 Aug 2017
Posts: 7
Own Kudos [?]: 0 [0]
Given Kudos: 22
Send PM
Intern
Intern
Joined: 28 Nov 2017
Posts: 16
Own Kudos [?]: 6 [1]
Given Kudos: 41
Location: India
Concentration: General Management
GPA: 3.16
WE:Engineering (Energy and Utilities)
Send PM
GMAT Club Verbal Expert
Joined: 13 Aug 2009
Status: GMAT/GRE/LSAT tutors
Posts: 6920
Own Kudos [?]: 63665 [0]
Given Kudos: 1773
Location: United States (CO)
GMAT 1: 780 Q51 V46
GMAT 2: 800 Q51 V51
GRE 1: Q170 V170

GRE 2: Q170 V170
Send PM
Intern
Intern
Joined: 28 Nov 2017
Posts: 16
Own Kudos [?]: 6 [0]
Given Kudos: 41
Location: India
Concentration: General Management
GPA: 3.16
WE:Engineering (Energy and Utilities)
Send PM
Re: An eyeglass manufacturer tried to boost sales for the summer [#permalink]
GMATNinja wrote:
gmatacer40 wrote:
VeritasPrepKarishma, GMATNinja, egmat
Why specifically option E is better than C?
My confusion is that option C suggests that the increase in the sales number of manufacturer would be less since 20% of small quantity (quantum of sales done by eligible Retailers) would be less and from the argument, it is evident that in the summer quarter, the quantum of sales was so much as to offset the discount effect.
Option E suggests that most of the retailers wont avail the manufacturer's discount in fall quarter as they already have inventories left.

Hence, my doubt is how option e has an edge over c?

Even if (C) is true, the manufacturer's plan already worked for the summer. So even though orders were unusually low last year summer, the plan still increased profits from those distributors. The same could be true in the fall. Distributors whose orders were low last fall might increase their orders because of the discount this fall.

Regardless of the number of orders, the plan can still cause a healthy gain in profits if sales are boosted by more than 20%.


GMATNinja
You have mentioned "So even though orders were unusually low last year summer". However, I am not able to deduce this specific information from the argument. According to me, if we take option C as true, it would talk about only fall quarter. So, we dont know whether orders were lw for summer quarter. please explain.
Current Student
Joined: 06 Feb 2016
Status:On the journey of achieving
Affiliations: Senior Manager, CA by profession, CFA(USA) Level 2
Posts: 254
Own Kudos [?]: 168 [0]
Given Kudos: 148
Location: India
Concentration: Finance, Finance
GMAT 1: 560 Q44 V23
GMAT 2: 530 Q39 V24
GMAT 3: 580 Q46 V24 (Online)
GMAT 4: 640 Q50 V26
GPA: 3.82
WE:Other (Commercial Banking)
Send PM
Re: An eyeglass manufacturer tried to boost sales for the summer [#permalink]
Experts mikemcgarry GMATNinja VeritasPrepKarishma egmat Please guide for this question.......I marked Option B as increased newspaper and advertising costs shall have a direct impact on net profit which is not considered in the argument.
VP
VP
Joined: 12 Feb 2015
Posts: 1065
Own Kudos [?]: 2103 [0]
Given Kudos: 77
Send PM
Re: An eyeglass manufacturer tried to boost sales for the summer [#permalink]
If the closing stock for summer quarter will become the opening stock for fall quarter then will the distributors stock more if they are already overstocked?.

Since it is mentioned in the question stem that "Many distributors qualified for this discount.", means many distributors might not avail the discount as they are already over stocked.

Option E is the correct choice!! "The distributors' ordering more goods in the summer quarter left them overstocked for the fall quarter".
Intern
Intern
Joined: 17 Apr 2018
Posts: 8
Own Kudos [?]: 1 [0]
Given Kudos: 30
Send PM
Re: An eyeglass manufacturer tried to boost sales for the summer [#permalink]
GMATNinja , @VeritasPrepKarishma , egmat - Request your help in understanding why the option E is better than option B in this question.

My thinking
B- If the manufacturer had put inputs other than the discounts then they might have had an impact on sales & hence we cannot assume that the same similar growth in sale will happen in fall quarter- given that it is specified that manufacturer plans to repeat the same discount but is not mentioned that other inputs will also be repeated
E- The distributors are overstocked for fall, but might choose to be even more overstocked. It might not necessarily result in failure of the plan.

Your help in understanding will be much appreciated.
Intern
Intern
Joined: 01 Apr 2017
Posts: 18
Own Kudos [?]: 13 [0]
Given Kudos: 6
Send PM
Re: An eyeglass manufacturer tried to boost sales for the summer [#permalink]
A) In general, a distributor's orders for the summer quarter are no higher than those for the spring quarter.
is not indicating any hint that will lead to paradox

(B) Along with offering special discounts to qualifying distributors, the manufacturer increased newspaper and radio advertising in those distributors' sales areas.
it means just probability of high sales is very high. but doesnot generate any doubt

(C) The distributors most likely to qualify for the manufacturer's special discount are those whose orders were unusually low a year earlier.
this is basically the same point mentioned in the passage. hence nothing imp

(D) The distributors who qualified for the manufacturer's special discount were free to decide how much of that discount to pass on to their own customers.
it doesnot relate to profit of manufacturer. hence not a relevant

(E) The distributors' ordering more goods in the summer quarter left them overstocked for the fall quarter.
if overstocked, there is a high chance that the price will fall. this will affect the net profit even if sales increases. hence
giving discount makes no sense to manufacturers!!
Intern
Intern
Joined: 25 Jun 2018
Posts: 40
Own Kudos [?]: 10 [0]
Given Kudos: 46
Send PM
An eyeglass manufacturer tried to boost sales for the summer [#permalink]
GMATNinja wrote:
[*] "sales" = "orders" in this argument.


GMATNinja

I did not think about this way. I thought "sales" is what customers buy (i.e. not what manufacturer sells to the distributors).
Hence, I eliminated E because customers will buy glasses in the fall, just like they bought glasses in summer (considering that distributors have enough glasses to sell. It does not matter whether glasses are overstocked or not. As long as distributors have glasses, they will sell them).

I also fell for B. I thought, advertisement was a factor that boosted the sales.
And, B also says that the manufacturer increased advertising (past tense), so I thought the manufacturer will not do advertisement in the fall, therefore the sales will not boost (I know that we can not know if the manufacturer will do advertisement in the fall or not. But this is a strengthen/weaken question. We do not look for an answer choice that is 100% certain, instead, we look for an answer choice that is likely to affect the argument. And that likelihood is "If the manufacturer will not do advertisement in the fall".

Any opinions?
avatar
Intern
Intern
Joined: 24 Sep 2021
Posts: 1
Own Kudos [?]: 0 [0]
Given Kudos: 0
Location: Anguilla
GMAT 1: 560 Q36 V52
GRE 1: Q162 V159
Send PM
Re: An eyeglass manufacturer tried to boost sales for the summer [#permalink]
manufacture concludes that fall quarter will have the same results as summer quarter.
GMAT Club Verbal Expert
Joined: 13 Aug 2009
Status: GMAT/GRE/LSAT tutors
Posts: 6920
Own Kudos [?]: 63665 [1]
Given Kudos: 1773
Location: United States (CO)
GMAT 1: 780 Q51 V46
GMAT 2: 800 Q51 V51
GRE 1: Q170 V170

GRE 2: Q170 V170
Send PM
Re: An eyeglass manufacturer tried to boost sales for the summer [#permalink]
1
Kudos
Expert Reply
custodio wrote:
GMATNinja wrote:
[*] "sales" = "orders" in this argument.


GMATNinja

I did not think about this way. I thought "sales" is what customers buy (i.e. not what manufacturer sells to the distributors).
Hence, I eliminated E because customers will buy glasses in the fall, just like they bought glasses in summer (considering that distributors have enough glasses to sell. It does not matter whether glasses are overstocked or not. As long as distributors have glasses, they will sell them).

I also fell for B. I thought, advertisement was a factor that boosted the sales.
And, B also says that the manufacturer increased advertising (past tense), so I thought the manufacturer will not do advertisement in the fall, therefore the sales will not boost (I know that we can not know if the manufacturer will do advertisement in the fall or not. But this is a strengthen/weaken question. We do not look for an answer choice that is 100% certain, instead, we look for an answer choice that is likely to affect the argument. And that likelihood is "If the manufacturer will not do advertisement in the fall".

Any opinions?

Taken out of context, the word "sales" could refer to either merchandise sold to a distributor, or merchandise sold by a distributor to a customer. So what is the intended meaning in this passage?

Quote:
An eyeglass manufacturer tried to boost sales for the summer quarter by offering its distributors a special discount if their orders for that quarter exceeded those for last year's summer quarter by at least 20 percent.

Here, the manufacturer is trying to boost its own sales. Who is the manufacturer selling things to? To its distributors -- who place orders with the manufacturer. So, the word "sales" must refer to glasses sold by the manufacturer to the distributor. The words "sales" and "orders" refer to the same exact thing in this passage.

Regarding answer choice (B): be careful about adding your own interpretations to answer choices. Another key aspect of critical reasoning is to always understand the answer choices in their exact words.

With that in mind, let's take a look at (B):

Quote:
(B) Along with offering special discounts to qualifying distributors, the manufacturer increased newspaper and radio advertising in those distributors' sales areas.

We know for certain that the manufacturer "increased newspaper and radio advertising" the previous summer, since this was done along with the discounts. As for this coming summer, based on their exact words, we have to conclude that we simply don't know if they'll advertise or not. It could be relevant to the question, but after reading answer choice (B), we simply don't have enough information.

I hope that helps!
User avatar
Non-Human User
Joined: 01 Oct 2013
Posts: 17218
Own Kudos [?]: 848 [0]
Given Kudos: 0
Send PM
Re: An eyeglass manufacturer tried to boost sales for the summer [#permalink]
Hello from the GMAT Club VerbalBot!

Thanks to another GMAT Club member, I have just discovered this valuable topic, yet it had no discussion for over a year. I am now bumping it up - doing my job. I think you may find it valuable (esp those replies with Kudos).

Want to see all other topics I dig out? Follow me (click follow button on profile). You will receive a summary of all topics I bump in your profile area as well as via email.
GMAT Club Bot
Re: An eyeglass manufacturer tried to boost sales for the summer [#permalink]
   1   2 
Moderators:
GMAT Club Verbal Expert
6920 posts
GMAT Club Verbal Expert
238 posts
CR Forum Moderator
832 posts

Powered by phpBB © phpBB Group | Emoji artwork provided by EmojiOne